0


Ich habe diese Gleichung mit Summen gegeben. Wenn Ich es soweit richtig verstehe wird \( \frac{1}{n} \) vorne einfach gekürzt. Allerdings weiß Ich nicht wie Ich auf den Zähler und Nenner auf der rechten Seite kommen soll. \( \bar{X} \) und \( \bar{Y} \) sind jewils gegeben als \( \frac{1}{n} \sum^n_{i=1} X_i \) und \( \frac{1}{n} \sum^n_{i=1} Y_i \). Ich glaube das genau diese Definitionen mir helfen würden aber Ich komm nicht genau drauf wieso die Gleichung bzw. Umformung so geht.

EDIT vom 13.11.2022 um 23:08:


Ich habe diese Gleichung mit Summen gegeben. Wenn Ich es soweit richtig verstehe wird 1n vorne einfach gekürzt. Allerdings weiß Ich nicht wie Ich auf den Zähler und Nenner auf der rechten Seite kommen soll.  und  sind jewils gegeben als 1n∑i=1nXi und 1n∑i=1nYi. Ich glaube das genau diese Definitionen mir helfen würden aber Ich komm nicht genau drauf wieso die Gleichung bzw. Umformung so geht.

Zusatz (aktuelle Rechnungen):

Diese Frage melden
gefragt

Student, Punkte: 16

 
Kommentar schreiben
1 Antwort
1
Rechne von rechts nach links. Schreibe vorher(!) $\frac1n$ in Zähler und Nenner (erweitere also damit). Dann rechne die Summen rechts aus (im Zähler ausmultiplizieren, im Nenner bin. Formel). Achte genauestens(!) auf die Summen. Beachte dann noch die beiden Def'n, die Du schon als hilfreich erkannt hast, sowie die Tatsache, dass $\sum\limits_{i=1}^n c =n\cdot c$ für konstantes $c$.
Und übrigens: Auf der linken Seite steht das $\bar X\,\bar Y$ bzw. das $(\bar X)^2$ nicht mit im Summenzeichen (heißt: darüber wird nicht summiert). Damit kommt alles prima hin.
Diese Antwort melden
geantwortet

Lehrer/Professor, Punkte: 38.9K

 

Vielen Dank für deine Antwort. Ich komme tatsächlich beim Zähler so definitiv weiter, nur komme Ich darauf, dass \( \bar{X}\bar{Y} \) trotzdem noch in der Summe sind. Da Ich vorher auf \( \sum_{i=1}^nX_iY_i -\sum_{i=1}^n\bar{X}\bar{Y} \) komme. Die beiden Terme in der Mitte kürzen sich ja raus. Beim Nenner komme Ich mit \( \sum_{i=1}^nX_i^2 - \sum_{i=1}^n2X_i\bar{X} +\sum_{i=1}^n\bar{X}^2 \) nicht weiter. Ich schätze mal Ich muss zu einem Punkt kommen wo Ich am ende \( \sum_{i=1}^nX_i^2 - 2\sum_{i=1}^nX_i^2 +\sum_{i=1}^n\bar{X}^2 \) habe?   ─   prossberg 13.11.2022 um 20:19

Dann hab Ich definitiv einiges verhauen :D. Weiß nicht genau warum Ich da voll auf dem Schlauch stehe. Ich habe meine Rechnungen hochgeladen.   ─   prossberg 13.11.2022 um 23:09

Tut mir leid Ich hatte gestern wenig Zeit und hab einfach das hochgeladen was Ich bis dahin (sehr schlecht und nachlässig) gemacht hatte. Habe es heute morgen nochmal sorgfältig mit deinen Tipps versucht und bin zum richtigen Ergebnis gekommen. Vielen Dank nochmal.   ─   prossberg 14.11.2022 um 10:49

Leider scheint diese Antwort Unstimmigkeiten zu enthalten und muss korrigiert werden. Mikn wurde bereits informiert.